Incorrect Answer explanation
I understand the correct answer, but I was hoping someone could help explain what the explanation...
mbshapir on February 6 at 01:41PM
  • December 2017 LSAT
  • SEC3
  • Q24
1
Reply
Why is it D
WHy would his companion ask what the tablet said, if he didnt read silently to himself. If I ...
Jvsquaq1 on January 16 at 01:32PM
  • December 2017 LSAT
  • SEC3
  • Q12
1
Reply
A vs. E
Please explain why E is a better answer choice than A, thank you!
Gabby_teixeira on June 26 at 11:51PM
  • December 2017 LSAT
  • SEC3
  • Q5
1
Reply
What about the middle ?
I know it's not always suggested to assume unless it's stated but there have been some l.S.A.T qu...
Nativeguy on June 6 at 11:50PM
  • December 2017 LSAT
  • SEC3
  • Q21
1
Reply
Please explain again
Can someone please explain to me why the correct answer is correct? Im having a hard time sket...
the66guy on October 27, 2022
  • December 2017 LSAT
  • SEC3
  • Q13
1
Reply
Confusion on Stimulus
I am very confused on this stimulus. It seems like there might be even multiple gaps....or I am j...
Sallyanne-Tejan on August 5, 2022
  • December 2017 LSAT
  • SEC3
  • Q16
1
Reply
Stimulus
For this question, is the stimulus asking for which is not necessarily true/not accurate? Need...
EmilyMarieMenendez on February 5, 2022
  • December 2017 LSAT
  • SEC3
  • Q12
4
Replies
Why is A Wrong
I understand how B is the correct answer but how do you know that A is incorrect / explains the p...
GabbyS on January 31, 2022
  • December 2017 LSAT
  • SEC3
  • Q22
2
Replies
Please explain
Can you please explain why B ic correct? If the temperature in Charlesville drops at night, w...
Sidra on August 4, 2021
  • December 2017 LSAT
  • SEC3
  • Q22
3
Replies
Question on answer choice D
Hi, so I am having a hard time working through answer choice D. I understand what others have sai...
SawyerJeppson on July 31, 2021
  • December 2017 LSAT
  • SEC3
  • Q21
1
Reply
December 2017 SEC 3 Q8
So, is the biologist claiming that island plants are going extinct at much faster rate because th...
kens on July 21, 2021
  • December 2017 LSAT
  • SEC3
  • Q8
3
Replies
How to...
Eliminate (B)? I had a hard time doing so. Thanks!
angelasargent on December 8, 2020
  • December 2017 LSAT
  • SEC3
  • Q17
2
Replies
I don't understand why D is correct
I don't understand this question or why D is correct
pgeorge2017 on December 8, 2020
  • December 2017 LSAT
  • SEC3
  • Q8
4
Replies
Why not E?
Through process of elimination, I got down to D and E, but neither seemed like a perfect fit to m...
Christy-Earls on September 7, 2020
  • December 2017 LSAT
  • SEC3
  • Q21
6
Replies
Why not C?
I don't understand why C couldn't also be correct
rwhittle@colgate.edu on July 5, 2020
  • December 2017 LSAT
  • SEC3
  • Q12
1
Reply
Explanation
Can someone please explain the right answer?
zia305 on June 24, 2020
  • December 2017 LSAT
  • SEC3
  • Q20
1
Reply
December 2017 SEC 3 Q6
What is the difference between A and B? Thanks in adavance!
kens on June 14, 2020
  • December 2017 LSAT
  • SEC3
  • Q6
1
Reply
December 2017 LSAT LG #13
Can someone please explain why A is incorrect? Thanks
kens on June 14, 2020
  • December 2017 LSAT
  • SEC3
  • Q13
1
Reply
December 2017 SEC 3 Q21
Can someone please explain why C is incorrect? Thanks in advance!
kens on June 14, 2020
  • December 2017 LSAT
  • SEC3
  • Q21
1
Reply
Explanation
Can someone please explain this to me? I am struggling to understand why the correct answer is th...
mjenei on June 13, 2020
  • December 2017 LSAT
  • SEC3
  • Q25
3
Replies